Difference between revisions of "2021 AMC 12B Problems/Problem 5"
(Created page with "The 2021 AMC 12B will be held on February 10th, 2021. The problems will not be made public until 24 hours after that.") |
Sugar rush (talk | contribs) |
||
Line 1: | Line 1: | ||
− | The | + | ==Problem== |
+ | The point <math>P(a,b)</math> in the <math>xy</math>-plane is first rotated counterclockwise by <math>90\deg</math> around the point <math>(1,5)</math> and then reflected about the line <math>y = -x</math>. The image of <math>P</math> after these two transformations is at <math>(-6,3)</math>. What is <math>b - a ?</math> | ||
+ | |||
+ | <math>\textbf{(A)} ~1 \qquad\textbf{(B)} ~3 \qquad\textbf{(C)} ~5 \qquad\textbf{(D)} ~7 \qquad\textbf{(E)} ~9</math> | ||
+ | ==Solution== | ||
+ | <math>\boxed{\textbf{(D)} ~7}</math> |
Revision as of 17:41, 11 February 2021
Problem
The point in the -plane is first rotated counterclockwise by around the point and then reflected about the line . The image of after these two transformations is at . What is
Solution